PANRE & PANRE-LA Intervention Complex Practice Exam

PANRE & PANRE-LA (Intervention Complex) Practice Exam

Congratulations - you have completed PANRE-LA Intervention Complex. You scored %%SCORE%% out of %%TOTAL%%. Your performance has been rated as %%RATING%%
Your answers are highlighted below.
Question 1
A 77 ­year ­old woman presents to the emergency department with the following signs and symptoms: dysarthria and dysphagia, vertigo, nausea, syncope, memory loss and disorientation, and ataxic gait. On physical examination, the patient has nystagmus, homonymous hemianopia, numbness in the area of the 12th cranial nerve, and facial weakness. You suspect a CVA. Which of the following arteries is most likely to be involved?
A
Middle cerebral artery
Hint:
The common presentation of an MCA stroke includes sudden onset of contralateral hemiparesis (weakness on one side of the body) and sensory loss, particularly in the face and arm. Other symptoms may include aphasia (difficulty with language), if the dominant hemisphere is affected, or neglect and spatial awareness difficulties if the non-dominant hemisphere is affected.
B
Posterior cerebral artery
Hint:
Posterior cerebral artery occlusion typically presents with visual field defects.
C
Vertebral-basilar artery
D
Anterior cerebral artery
Hint:
Anterior cerebral artery occlusion typically presents with contralateral hemiparesis with loss of sensibility in the foot and lower extremity, sometimes with urinary incontinence.
E
Posterior inferior cerebellar artery
Hint:
Posterior inferior cerebellar artery occlusion typically presents with ipsilateral ataxia and Horner’s syndrome.
Question 1 Explanation: 
This patient has had a CVA involving the vertebral-basilar system. The vertebral-basilar artery supplies blood to the brainstem and cerebellum. The signs and symptoms of a vertebral-basilar stroke are (1) dysarthria and dysphagia; (2) vertigo, nausea, and vomiting; (3) disorientation; (4) ataxic gait (ipsilateral cerebellar ataxia); (5) visual symptoms (double vision and blurred vision); (6) dysphagia; (7) ocular signs (nystagmus, conjugate gaze paralysis, and ophthalmoplegia); (8) akinetic mutism (locked¬in syndrome when basilar artery occlusion occurs); (9) numbness of lips and face; (10) facial weakness, alternating motor paresis; and (11) drop attacks, syncope (Doppler studies can detect vertebrobasilar embolic sources). (Review PANRE Blueprint Topic: Stroke)
Question 2
A 65-year-old man with a history of chronic obstructive pulmonary disease (COPD) and diabetes mellitus presents to the clinic with fever, productive cough, and dyspnea for 3 days. He has been smoking half a pack of cigarettes per day for 40 years. On physical examination, he has a temperature of 38.5°C (101.3°F), heart rate of 110 beats per minute, blood pressure of 150/90 mmHg, respiratory rate of 24 breaths per minute, and oxygen saturation of 92% on room air. He has decreased breath sounds and crackles at the right lower lobe. His chest X-ray shows a right lower lobe consolidation. Which of the following organisms is most likely responsible for this patient’s pneumonia?
A
Streptococcus pneumoniae
B
Mycoplasma pneumoniae
Hint:
Mycoplasma pneumoniae is a cause of atypical pneumonia that affects younger adults or children. It usually occurs in outbreaks in close-contact settings such as schools or military barracks. It presents with gradual onset of low-grade fever, dry cough, headache, and malaise. The physical examination may be normal or reveal scattered crackles. The chest X-ray may show interstitial infiltrates or patchy areas of consolidation. The diagnosis can be made by serology or polymerase chain reaction (PCR). The treatment is macrolides or doxycycline.
C
Pseudomonas aeruginosa
Hint:
Pseudomonas aeruginosa is a cause of hospital-acquired or ventilator-associated bacterial pneumonia that affects immunocompromised patients or those with structural lung disease such as cystic fibrosis. It presents with high fever, productive cough with greenish sputum, hemoptysis, and dyspnea. The physical examination may reveal signs of consolidation similar to S.pneumoniae but also cyanosis and clubbing due to chronic hypoxia. The chest X-ray may show multifocal infiltrates with cavitation. The diagnosis can be made by sputum culture showing gram-negative rods that produce blue-green pigment and have a fruity odor. The treatment is broad-spectrum antibiotics such as piperacillin-tazobactam or cefepime plus an aminoglycoside.
D
Legionella pneumophila
Hint:
Legionella pneumophila is a cause of atypical pneumonia that affects older adults or those with underlying medical conditions such as COPD, diabetes, renal failure, malignancy, or immunosuppression. It usually occurs after exposure to contaminated water sources such as air conditioners, cooling towers, showers, fountains, etc. It presents with high fever, chills, cough, dyspnea, and gastrointestinal symptoms such as diarrhea, nausea, vomiting, and abdominal pain. The physical examination may reveal signs of consolidation similar to S.pneumoniae but also relative bradycardia and hyponatremia due to inappropriate antidiuretic hormone secretion. The chest X-ray may show patchy infiltrates that involve both lungs. The diagnosis can be made by urine antigen test, sputum culture, or PCR.
E
Staphylococcus aureus
Hint:
Staphylococcus aureus is not the most likely cause of pneumonia in this patient. The most common cause of bacterial pneumonia is Streptococcus pneumoniae (pneumococcus), especially in patients with underlying chronic conditions such as COPD and diabetes mellitus. Staphylococcus aureus pneumonia is more often seen in young children, post-viral infections, or hospital-associated settings. It can also cause severe complications such as necrotizing pneumonia, bacteremia, or sepsis.
Question 2 Explanation: 
Streptococcus pneumoniae is the most common cause of community-acquired bacterial pneumonia. It usually affects older adults or those with underlying medical conditions such as COPD or diabetes. It typically presents with acute onset of fever, cough, dyspnea, and pleuritic chest pain. The sputum may be rusty-colored or purulent. The physical examination may reveal signs of consolidation such as decreased breath sounds, crackles, dullness to percussion, and increased tactile fremitus. The chest X-ray may show lobar or segmental infiltrates. The diagnosis can be confirmed by blood culture, sputum culture, or urine antigen test The treatment is usually empiric antibiotics such as penicillin or macrolides depending on the local resistance patterns. (Review PANRE Blueprint Topic: Penumonia)
Question 3
A 45-year-old woman presents to the emergency department with acute onset of dyspnea, chest pain, and palpitations. She has a history of breast cancer treated with chemotherapy six months ago. She denies any fever, cough, hemoptysis, or leg swelling. On physical examination, she is tachypneic (respiratory rate 28 breaths per minute), tachycardic (heart rate 120 beats per minute), hypotensive (blood pressure 90/60 mmHg), and hypoxic (oxygen saturation 88% on room air). Her lungs are clear to auscultation bilaterally. Her heart sounds are normal with no murmurs or gallops. Her abdomen is soft and nontender. Her extremities are warm and well perfused with no edema or tenderness. Which of the following is the most appropriate next step in management?
A
Order a D-dimer test
Hint:
Ordering a D-dimer test is not appropriate in this case because it has low specificity for PE and may be elevated in other conditions such as malignancy or inflammation. A negative D-dimer test can help rule out PE in patients with low clinical probability, but this patient has a high clinical probability. B) Ordering a chest X-ray is not appropriate in this case because it has low sensitivity for PE and may be normal or show nonspecific findings such as atelectasis or pleural effusion. A chest X-ray may be useful to rule out other causes of dyspnea such as pneumonia or pneumothorax, but it cannot confirm or exclude PE.
B
Order a chest X-ray
Hint:
Ordering a chest X-ray is not appropriate in this case because it has low sensitivity for PE and may be normal or show nonspecific findings such as atelectasis or pleural effusion. A chest X-ray may be useful to rule out other causes of dyspnea such as pneumonia or pneumothorax, but it cannot confirm or exclude PE.
C
Order a computed tomography pulmonary angiography (CTPA)
Hint:
Ordering a CTPA is the preferred diagnostic test for PE in most cases because it has high sensitivity and specificity for detecting pulmonary artery occlusion. However, in patients with hemodynamic instability due to massive PE, CTPA may cause further delay in treatment and worsen outcomes. Therefore, CTPA should be deferred until after thrombolytic therapy is initiated.
D
Start anticoagulation therapy
Hint:
Starting anticoagulation therapy is an essential part of management for all patients with suspected or confirmed PE because it prevents further clot formation and propagation. However, anticoagulation alone may not be sufficient to dissolve large clots that cause massive PE and hemodynamic compromise. Therefore, anticoagulation should be combined with thrombolytic therapy in these cases.
E
Start thrombolytic therapy
Question 3 Explanation: 
The patient has a high clinical suspicion for pulmonary embolism (PE) based on her symptoms, vital signs, and history of malignancy. She also has signs of hemodynamic instability (tachycardia, hypotension, hypoxia), which indicate a massive PE that may cause right ventricular failure and cardiogenic shock. In such cases, immediate thrombolytic therapy is indicated to dissolve the clot and restore pulmonary blood flow. Thrombolytic therapy has been shown to reduce mortality in patients with massive PE. (Review PANRE Blueprint Topic: Pulmonary Embolism (PE))
Question 4

A 42-year-old woman with a history of GERD presents to your clinic for follow-up. She has been taking omeprazole 20 mg daily for the past 6 months and reports significant improvement in her heartburn and regurgitation symptoms. She has also made lifestyle modifications such as avoiding spicy and fatty foods, quitting smoking, and elevating the head of her bed. She asks you if she can stop taking omeprazole or reduce the dose. What is the most appropriate next step in management?

A
Continue omeprazole 20 mg daily indefinitely
Hint:
This option may expose the patient to unnecessary risks of long-term PPI use without attempting a trial of dose reduction or discontinuation.
B
Discontinue omeprazole and monitor symptoms
Hint:
This option may cause rebound acid hypersecretion and recurrence of GERD symptoms due to abrupt withdrawal of PPI therapy.
C
Switch to famotidine 20 mg twice daily
Hint:
Immediately switching to an H2 blocker (such as famotidine) may not provide adequate symptom control for GERD, as H2 blockers are less potent than PPIs in suppressing gastric acid secretion. Slowly taper off the PPI first over 2-4 weeks (the higher the dose, the longer the taper). If symptoms return, it would be appropriate to start again with an H2 blocker.  If long-term treatment is needed, H2 blockers allow better absorption of nutrients than PPIs and so potentially have fewer long-term adverse effects. If symptoms are still difficult to control, consider adding the PPI back at the lowest effective dose. 
D
Taper omeprazole to every other day for 4 weeks
E
Perform an upper endoscopy
Hint:
This option is not indicated for patients with uncomplicated GERD who have responded well to medical therapy. Endoscopy is reserved for patients who have alarm features such as dysphagia, odynophagia, weight loss, and anemia.
Question 4 Explanation: 
The patient has a history of GERD that has responded well to PPI therapy and lifestyle modifications. The goal of treatment is to achieve symptom relief and prevent complications such as esophagitis, stricture, Barrett's esophagus, or adenocarcinoma. PPIs are more effective than H2 blockers or antacids for healing erosive esophagitis and maintaining remission. However, long-term use of PPIs may be associated with adverse effects such as increased risk of fractures, infections, hypomagnesemia, vitamin B12 deficiency, and chronic kidney disease. Therefore, it is reasonable to attempt a step-down approach after achieving symptom control with PPIs for at least 8 weeks. This involves tapering the dose of PPI gradually over 2-4 weeks to avoid rebound acid hypersecretion. If symptoms recur after discontinuation of PPIs, then switching to an H2 blocker or restarting PPI maintenance therapy with the lowest effective dose may be indicated. (Review PANRE Blueprint Topic: GERD)
Question 5

A 35-year-old man who recently moved from New York to Arizona presents to the clinic with fever, cough, chest pain, and weight loss for the past two weeks. He has no significant past medical history and does not smoke or use drugs. He works as a construction worker and has been exposed to dust at his job site. His vital signs are normal except for a temperature of 38.2°C (100.8°F). His physical examination reveals crackles in the right lower lung field. A chest radiograph shows a right lower lobe consolidation with a small cavity. What is the most likely causative organism of this patient's condition?

A
Mycobacterium tuberculosis
Hint:
Mycobacterium tuberculosis is incorrect because it is a bacterial infection that causes chronic granulomatous inflammation of the lungs and other organs. Tuberculosis can present similarly to coccidioidomycosis but it is less common in the United States and more likely to affect immunocompromised patients or those with risk factors such as exposure to endemic areas or infected individua]. Tuberculosis can be diagnosed by sputum acid-fast bacilli smear and culture or nucleic acid amplification test.
B
Coccidioides immitis
C
Histoplasma capsulatum
Hint:
Histoplasma capsulatum is incorrect because it is another systemic fungal infection that causes pulmonary and extrapulmonary manifestations similar to coccidioidomycosis but it is endemic in regions along the Ohio and Mississippi River valleys rather than in arid regions like Arizona. Histoplasma capsulatum exists as yeast in tissue rather than spherules like Coccidioides immitis. Histoplasma capsulatum can be diagnosed by urine antigen test or fungal culture from respiratory specimens.
D
Blastomyces dermatitidis
Hint:
Blastomyces dermatitidis is incorrect because it is another systemic fungal infection that causes pulmonary and extrapulmonary manifestations similar to coccidioidomycosis but it is endemic in regions around the Great Lakes and Mississippi River rather than in arid regions like Arizona. Blastomyces dermatitidis also exists as yeast in tissue rather than spherules like Coccidioides immitis. Blastomyces dermatitidis can be diagnosed by fungal culture from respiratory specimens or skin biopsy.
E
Staphylococcus aureus
Hint:
Staphylococcus aureus is incorrect because it is a bacterial infection that can cause secondary pneumonia following viral respiratory infections such as influenza. Staphylococcus aureus pneumonia typically affects patients with underlying lung diseases such as cystic fibrosis or chronic obstructive pulmonary disease. Staphylococcus aureus pneumonia can cause]necrotizing pneumonia with multiple cavities on chest radiograph. Staphylococcus aureus can be diagnosed by sputum Gram stain and culture or blood culture.
Question 5 Explanation: 
The patient has signs and symptoms of coccidioidomycosis, which is a systemic fungal infection caused by Coccidioides immitis or Coccidioides posadasii. These fungi are endemic in arid regions of the southwestern United States (such as Arizona), Mexico, Central America, and South America. They exist as mycelia in soil that can become airborne when disturbed by wind or human activities such as construction. Inhalation of spores can lead to primary pulmonary infection that may manifest as pneumonia with fever, cough, chest pain, hemoptysis, and weight loss. Chest radiograph may show infiltrates, nodules, cavities, or pleural effusions. Most cases are self-limited but some patients may develop chronic pulmonary disease or disseminated disease involving skin (erythema nodosum), bones (osteomyelitis), joints (arthritis), meninges (meningitis), or other organs. (Review PANRE Blueprint Topic: Fungal Pneumonia)
Question 6
A 65-year-old man with a history of type 2 diabetes mellitus presents to the emergency department with altered mental status, polyuria, and polydipsia. He has been feeling unwell for the past week with a urinary tract infection that he has been self-treating with cranberry juice. His vital signs are: blood pressure 180/100 mmHg, heart rate 110 beats per minute, respiratory rate 24 breaths per minute, temperature 37.8°C (100°F), and oxygen saturation 95% on room air. His physical examination reveals dry mucous membranes, poor skin turgor, and decreased level of consciousness. His laboratory tests show:
  • Serum glucose: 900 mg/dL
  • Serum sodium: 150 mEq/L
  • Serum potassium: 4.0 mEq/L
  • Serum bicarbonate: 18 mEq/L
  • Serum osmolality: 350 mOsm/kg
  • Urine ketones: negative
Which of the following is the most appropriate initial treatment?
A
Intravenous insulin infusion
Hint:
Insulin infusion is not the first-line treatment for HHS, as it can worsen dehydration, hypokalemia, and cerebral edema if given before adequate fluid replacement. Insulin should be started after fluid resuscitation at a low dose to avoid rapid drops in serum glucose and osmolality.
B
Intravenous normal saline infusion
C
Intravenous sodium bicarbonate infusion
Hint:
Sodium bicarbonate infusion is not indicated for HHS unless there is severe acidosis (pH <7.0). The patient’s serum bicarbonate level is mildly low (18 mEq/L), but not enough to warrant bicarbonate therapy. Moreover, sodium bicarbonate can increase serum osmolality and worsen cerebral edema.
D
Intravenous potassium chloride infusion
Hint:
Intravenous potassium chloride infusion is unnecessary for HHS unless there is hypokalemia (<3.5 mEq/L). The patient’s serum potassium level is normal (4.0 mEq/L), so potassium supplementation is not required. However, potassium levels should be monitored closely during fluid and insulin therapy, as they may drop rapidly due to intracellular shifts.
E
Subcutaneous insulin glargine injection
Hint:
Insulin glargine injection is a long-acting insulin that provides basal coverage for up to 24 hours. It is not suitable for treating acute hyperglycemia in HHS, as it has a slow onset of action and cannot be titrated easily according to blood glucose levels. Moreover, subcutaneous insulin administration may be unreliable in patients with poor perfusion due to dehydration.
Question 6 Explanation: 
This patient has hyperosmolar hyperglycemic syndrome (HHS), which is a complication of diabetes mellitus characterized by severe hyperglycemia (>600 mg/dL), hyperosmolality (>320 mOsm/kg), and dehydration in the absence of ketoacidosis. HHS is often triggered by an acute stressor such as infection, medication noncompliance, or excessive carbohydrate intake. The mainstay of treatment for HHS is fluid replacement with isotonic saline to correct dehydration, lower serum glucose and osmolality, and improve renal function. Insulin therapy can be initiated after adequate fluid resuscitation, usually at a low dose (0.05-0.1 units/kg/hour). Electrolyte abnormalities such as hypokalemia or acidosis should be corrected as needed.
Question 7
A 45-year-old male presents to his primary care PA with complaints of increased abdominal girth and elevated blood pressure. On examination, his body mass index (BMI) is 32 kg/m2, waist circumference is 102 cm, and blood pressure is 140/90 mmHg. Laboratory studies reveal fasting glucose of 100 mg/dL, triglycerides of 200 mg/dL, and high-density lipoprotein (HDL) cholesterol of 35 mg/dL. Which of the following is the most appropriate initial intervention for this patient?
A
Prescribe lifestyle modification focused on weight reduction and increased physical activity
B
Initiate metformin therapy for hyperglycemia
Hint:
Metformin therapy is indicated for hyperglycemia in patients with type 2 diabetes. However, in this patient, the fasting glucose is only mildly elevated and does not meet the criteria for diabetes.
C
Prescribe a statin for hyperlipidemia
Hint:
Statin therapy is indicated for patients with hyperlipidemia and an increased risk of cardiovascular disease. However, in this patient, the LDL cholesterol level is not reported, and the primary intervention should be lifestyle modifications.
D
Start antihypertensive therapy with a thiazide diuretic
Hint:
Antihypertensive therapy is indicated for patients with elevated blood pressure. However, lifestyle modifications should be attempted first before initiating medication therapy.
E
Refer the patient for bariatric surgery
Hint:
Bariatric surgery is indicated for patients with severe obesity (BMI ≥40 kg/m2) or BMI ≥35 kg/m2 with significant comorbidities. Although the patient in the scenario has a BMI of 32 kg/m2, the primary intervention should be lifestyle modifications before considering bariatric surgery.
Question 7 Explanation: 
The patient in the scenario meets the criteria for metabolic syndrome based on the presence of central obesity (waist circumference >102 cm), elevated blood pressure (≥130/85 mmHg), elevated triglycerides (≥150 mg/dL), low HDL cholesterol (<40 mg/dL in men), and impaired fasting glucose (≥100 mg/dL). Aggressive lifestyle modification focused on weight reduction and increased physical activity is the primary therapy for the management of metabolic syndrome. Lifestyle modifications can improve all the components of metabolic syndrome and reduce the risk of cardiovascular disease. (Review PANRE Blueprint Topic: Metabolic Syndrome)
Question 8
A 65-year-old woman with a history of atrial fibrillation, hypertension, and obesity presents to the emergency department with acute onset of severe left leg pain and swelling. She has been taking warfarin for anticoagulation but admits to missing some doses in the past week. Her blood pressure is 180/100 mmHg, pulse is 110 beats/min and irregular, and respiratory rate is 20 breaths/min. Her physical examination reveals a warm, tender, erythematous left lower extremity with prominent superficial veins and a positive Homan’s sign. Her international normalized ratio (INR) is 1.5. What is the most appropriate next step in the management of this patient?
A
Start heparin infusion and adjust warfarin dose
Hint:
Heparin infusion and warfarin dose adjustment are indicated for patients with confirmed deep vein thrombosis (DVT) who have subtherapeutic INR levels. However, this patient has not yet been diagnosed with DVT and needs further diagnostic testing before initiating anticoagulation therapy.
B
Order duplex ultrasonography of the lower extremities
C
Administer tissue plasminogen activator (tPA)
Hint:
tPA is a thrombolytic agent that dissolves blood clots by activating plasminogen into plasmin. Plasmin degrades fibrin, which forms the meshwork of clots. tPA may be used for patients with massive or life-threatening DVT who have contraindications to anticoagulation or who have failed anticoagulation therapy. However, it carries a high risk of bleeding complications and should not be used without confirming DVT diagnosis first.
D
Perform venography of the left leg
Hint:
Venography is an invasive procedure that involves injecting contrast dye into a vein and taking X-ray images to visualize any obstruction or narrowing in the venous system. It was once considered the gold standard for diagnosing DVT but has been largely replaced by duplex ultrasonography due to its higher cost, invasiveness, discomfort, and risk of allergic reactions or nephrotoxicity from contrast dye.
E
Apply compression stockings and elevate the leg
Hint:
Compression stockings are elastic garments that apply graduated pressure on the legs to improve venous return and prevent edema formation. They may be used as an adjunctive treatment for patients with confirmed DVT, along with anticoagulation therapy. Elevation of the leg can also reduce swelling and pain by decreasing hydrostatic pressure in the veins. However, these interventions do not address the underlying cause of DVT or prevent clot propagation or embolization. They should not be used as the sole therapy for suspected or confirmed DVT without diagnostic testing or anticoagulation therapy.
Question 8 Explanation: 
Duplex ultrasonography is a non-invasive test that uses sound waves to visualize the blood flow in the veins of the legs. It can detect thrombi by measuring the diameter, compressibility, and flow characteristics of the veins. It has high sensitivity and specificity for diagnosing DVT. It is considered the first-line diagnostic test for patients with suspected DVT. If negative, it can be repeated in one week or combined with D-dimer testing to rule out DVT. (Review PANRE Blueprint Topic: Deep venous thrombosis)
Question 9
A 55-year-old male with a history of hypertension and diabetes presents to his primary care PA complaining of fatigue, dyspnea on exertion, and edema in his lower extremities. Echocardiogram reveals an ejection fraction of 25% and left ventricular dilation. Which of the following medications is recommended for the initial treatment of this patient's systolic heart failure?
A
Spironolactone
Hint:
Spironolactone is a potassium-sparing diuretic that is used in patients with advanced heart failure to reduce the risk of hospitalization and improve survival.
B
Digoxin
Hint:
Digoxin is a cardiac glycoside that is used to control heart rate in patients with heart failure and atrial fibrillation but does not improve mortality.
C
Furosemide
Hint:
Furosemide is a loop diuretic that is used to treat volume overload in patients with heart failure but does not improve mortality.
D
Lisinopril
E
Verapamil
Hint:
Verapamil is a calcium channel blocker that is contraindicated in systolic heart failure due to its negative inotropic effects.
Question 9 Explanation: 
The initial treatment of systolic heart failure with reduced ejection fraction includes angiotensin-converting enzyme (ACE) inhibitors or angiotensin receptor blockers (ARBs) unless contraindicated. In this patient, lisinopril, an ACE inhibitor, is the preferred medication. ACE inhibitors decrease the risk of mortality and hospitalization and improve symptoms in patients with systolic heart failure. Spironolactone is a potassium-sparing diuretic that is used in patients with advanced heart failure to reduce the risk of hospitalization and improve survival. Digoxin is a cardiac glycoside that is used to control heart rate in patients with heart failure and atrial fibrillation but does not improve mortality. Furosemide is a loop diuretic that is used to treat volume overload in patients with heart failure but does not improve mortality. Verapamil is a calcium channel blocker that is contraindicated in systolic heart failure due to its negative inotropic effects. (Review PANRE Blueprint Topic: Heart Failure)
Question 10
A 72-year-old woman with a history of hypertension, diabetes mellitus, and atrial fibrillation presents to the emergency department with sudden onset of left-sided weakness and slurred speech. She was last seen normal 2 hours ago by her daughter. Her vital signs are: blood pressure 180/100 mmHg, pulse 110 beats/min irregularly irregular, respiratory rate 18 breaths/min, temperature 36.5°C (97.7°F), and oxygen saturation 98% on room air. On physical examination, she has left facial droop, left hemiparesis (4/5 strength), and dysarthria. Her NIH Stroke Scale score is 8. A non-contrast head CT scan shows no evidence of hemorrhage. What is the most appropriate next step in management?
A
Administer intravenous alteplase
B
Administer intravenous heparin
Hint:
Intravenous heparin administration is not recommended for acute ischemic stroke because it does not improve outcomes and may increase the risk of bleeding complications.
C
Perform carotid endarterectomy
Hint:
Carotid endarterectomy is a surgical procedure that removes plaque from the carotid artery to prevent recurrent strokes in patients with significant carotid stenosis (>70%) who are asymptomatic or have had a transient ischemic attack (TIA) or minor stroke within the past 6 months. It is not indicated for patients with acute ischemic stroke who have not been stabilized medically.
D
Perform mechanical thrombectomy
Hint:
Mechanical thrombectomy is an endovascular procedure that uses a catheter-based device to remove large vessel occlusions in patients with acute ischemic stroke who have a large penumbra (area of potentially salvageable brain tissue). It can be performed within 24 hours of symptom onset if intravenous alteplase is contraindicated or ineffective. However, it should not delay or replace intravenous alteplase if eligible.
E
Start oral aspirin
Hint:
Oral aspirin is recommended for secondary prevention of ischemic stroke after initial treatment with intravenous alteplase or mechanical thrombectomy. It can also be used as initial treatment for patients who present more than 24 hours after symptom onset or who have contraindications to intravenous alteplase. However, it should not be given within 24 hours of receiving intravenous alteplase because it may increase the risk of bleeding complications.
Question 10 Explanation: 
The patient has a clinical diagnosis of acute ischemic stroke, which is caused by occlusion of a cerebral artery by a thrombus or embolus. The most important factor in determining the treatment of acute ischemic stroke is the time from symptom onset to presentation. If the patient presents within 4.5 hours of symptom onset and has no contraindications, intravenous alteplase (a tissue plasminogen activator or tPA) should be administered as soon as possible to dissolve the clot and restore blood flow to the ischemic brain tissue. Intravenous alteplase has been shown to improve functional outcomes and reduce disability after acute ischemic stroke. The patient meets the criteria for intravenous alteplase because she presented within 4.5 hours of symptom onset, has no evidence of hemorrhage on head CT scan, and has no other contraindications such as recent surgery, active bleeding, severe hypertension (>185/110 mmHg), or use of anticoagulants. The other answer choices are incorrect because they are not indicated or effective in this scenario. (Review PANRE Blueprint Topic: Stroke)
Question 11
A 32-year-old female presents to the clinic with a two-month history of depressed mood, decreased energy, difficulty concentrating, and feelings of hopelessness. She denies any history of substance abuse, manic or hypomanic episodes, or psychotic symptoms. She has no history of suicidal ideation or attempts. What is the most appropriate initial treatment for this patient?
A
Cognitive-behavioral therapy (CBT)
Hint:
Cognitive-behavioral therapy (CBT) is a psychotherapy technique that can be used as an adjunct to pharmacotherapy in the treatment of major depressive disorder. However, it is not the most appropriate initial treatment for the patient in this case.
B
Fluoxetine
C
Electroconvulsive therapy (ECT)
Hint:
Electroconvulsive therapy (ECT) is reserved for patients who have not responded to pharmacotherapy or who are experiencing severe symptoms such as suicidal ideation. It is not the most appropriate initial treatment for the patient in this case.
D
Lithium
Hint:
Lithium is used in the treatment of bipolar disorder and is not indicated for the treatment of major depressive disorder.
E
Mindfulness-based stress reduction (MBSR)
Hint:
Mindfulness-based stress reduction (MBSR) is a non-pharmacological approach that can be used as an adjunct to pharmacotherapy in the treatment of major depressive disorder. However, it is not the most appropriate initial treatment for the patient in this case.
Question 11 Explanation: 
The patient's symptoms are consistent with a major depressive disorder, which is characterized by a depressed mood, loss of interest or pleasure, significant weight loss or gain, insomnia or hypersomnia, psychomotor agitation or retardation, fatigue or loss of energy, feelings of worthlessness or excessive guilt, diminished ability to think or concentrate, and recurrent thoughts of death or suicide. The most appropriate initial treatment for major depressive disorder is pharmacotherapy with selective serotonin reuptake inhibitors (SSRIs), such as fluoxetine. SSRIs are first-line agents for the treatment of major depressive disorder due to their efficacy and tolerability. They are generally well-tolerated and have a lower risk of side effects compared to other antidepressants. (Review PANRE Blueprint Topic: Major depressive disorder)
Question 12
A 25-year-old woman presents to her primary care physician with complaints of nasal congestion, sneezing, and watery eyes for the past two weeks. She says her symptoms worsen in the morning and improve throughout the day. She also notices that her symptoms get worse when she visits her parents who have a cat. She has no history of asthma, eczema, or sinusitis. On physical examination, she has clear nasal discharge, pale nasal mucosa, and conjunctival injection. Which of the following is the most appropriate next step in diagnosing this patient?
A
Nasal endoscopy
Hint:
Nasal endoscopy is a procedure that involves inserting a flexible tube with a camera into the nose to visualize the nasal cavity and sinuses. It may be useful for diagnosing conditions such as nasal polyps, tumors, foreign bodies, or fungal infections. However, it is not necessary for diagnosing allergic rhinitis which can be based on history and physical examination alone.
B
Skin prick testing
C
Serum IgE level
Hint:
Serum IgE level is a blood test that measures the amount of immunoglobulin E (IgE), an antibody that mediates allergic reactions. It may be elevated in patients with allergic rhinitis but it is not specific for any particular allergen. Therefore, it cannot help identify what causes allergic rhinitis in this patient.
D
Nasal smear for eosinophils
Hint:
Nasal smear for eosinophils is a test that involves collecting a sample of nasal secretions and staining it for eosinophils, a type of white blood cell that is involved in allergic inflammation. It may be positive in patients with allergic rhinitis but it is not specific for any particular allergen. Therefore, it cannot help identify what causes allergic rhinitis in this patient.
E
CT scan of sinuses
Hint:
CT scan of sinuses is an imaging test that uses X-rays to create detailed pictures of the sinuses. It may be useful for diagnosing conditions such as chronic sinusitis, nasal polyps, tumors, or fungal infections. However, it is unnecessary for diagnosing allergic rhinitis, which can be based on history and physical examination alone.
Question 12 Explanation: 
Skin prick testing is the preferred diagnostic test to provide evidence of an allergic cause of the patient’s symptoms. It involves applying small amounts of allergens to the skin and observing for any reactions, such as wheels or flares. It can help identify specific allergens that trigger allergic rhinitis and guide treatment options such as avoidance or immunotherapy. (Review PANRE Blueprint Topic: Rhinitis)
Question 13
A 4-year-old girl is brought to the otolaryngologist by her mother because of recurrent ear infections and hearing loss. She has had six episodes of acute otitis media in the past year and has persistent fluid in her middle ears. Her speech development is delayed and she often complains of ear pain and pressure. Audiometry shows a bilateral conductive hearing loss of 30 dB. Which of the following is the most appropriate treatment for this patient?
A
Antibiotics
Hint:
Antibiotics are incorrect because they are not effective for OME unless there is a superimposed acute infection.
B
Antihistamines
Hint:
Antihistamines are incorrect because they have no proven benefit for OME and may cause sedation and dryness.
C
Decongestants
Hint:
Decongestants are incorrect because they have no proven benefit for OME and may cause adverse effects such as hypertension, insomnia, and irritability.
D
Steroids
Hint:
Steroids are incorrect because they have no proven benefit for OME and may cause systemic side effects such as weight gain, immunosuppression, and growth retardation.
E
Tympanostomy tubes
Question 13 Explanation: 
The patient has chronic otitis media with effusion (OME), which is a persistent accumulation of fluid in the middle ear without signs of infection. OME can cause hearing loss, speech delay, and discomfort. The main indication for tympanostomy tubes (also known as ventilation tubes or grommets) is OME that persists for more than three months with significant hearing loss or speech impairment. Tympanostomy tubes are small plastic or metal tubes that are inserted into the tympanic membrane to allow air to enter the middle ear and drain the fluid. (Review PANRE Blueprint Topic: Acute and Chronic Otitis Media)
Question 14
A 55-year-old woman with a history of smoking presents to her primary care PA with complaints of chronic cough productive of white sputum, dyspnea on exertion, and frequent respiratory infections. She has no history of asthma or allergies. Her vital signs are normal except for a respiratory rate of 24 breaths per minute. On physical examination, she has a prolonged expiratory phase, wheezes, and crackles at both lung bases. Her chest radiograph shows increased bronchovascular markings and flattened diaphragms. Her pulmonary function tests show an FEV1/FVC ratio of 0.6, an increased residual volume, and an FEV1 of 70% predicted. Which of the following is the stage of her chronic bronchitis according to the GOLD system?
A
Mild
Hint:
This stage would require an FEV1 ≥ 80% predicted, which is higher than the patient’s value.
B
Moderate
C
Severe
Hint:
This stage would require an FEV1 30-49% predicted, which is lower than the patient’s value.
D
Very severe
Hint:
This stage would require an FEV1 < 30% predicted, which is much lower than the patient’s value.
E
End-stage
Hint:
This is not a valid stage according to the GOLD system. It may refer to a terminal condition that requires palliative care or hospice.
Question 14 Explanation: 
The patient has chronic bronchitis, which is a type of chronic obstructive pulmonary disease (COPD) characterized by chronic productive cough for at least 3 months in each of 2 consecutive years. The GOLD system uses the FEV1 test from the pulmonary function test to categorize the severity of COPD into stages. The FEV1 means the amount of air you can forcefully exhale in 1 second. The stages of COPD, according to the GOLD system, are:
  • Mild: FEV1 ≥ 80% predicted
  • Moderate: 50% ≤ FEV1 < 80% predicted
  • Severe: 30% ≤ FEV1 < 50% predicted
  • Very severe: FEV1 < 30% predicted
The patient has an FEV1 of 70% predicted, which falls into the moderate stage. She also has other features of COPD such as dyspnea on exertion, frequent respiratory infections, prolonged expiratory phase, wheezes, crackles, increased bronchovascular markings, and flattened diaphragms. (Review PANRE Blueprint Topic: Chronic Bronchitis)
Question 15
A 35-year-old woman presents to her primary care provider with complaints of chronic worry and nervousness for the past 8 months. She says she worries about everything, such as her health, her family, her work, and her finances. She has difficulty sleeping, concentrating, and relaxing. She also experiences muscle tension, headaches, and palpitations. She denies any history of trauma, substance abuse, or other psychiatric disorders. She has no medical problems and takes no medications. Her vital signs are normal and her physical examination is unremarkable. Which of the following is the most likely diagnosis?
A
Panic disorder
Hint:
Panic disorder is characterized by recurrent unexpected panic attacks that cause fear of having another attack or avoidance of situations that might trigger an attack. Panic attacks are sudden episodes of intense fear or discomfort that peak within minutes and are accompanied by at least 4 physical or cognitive symptoms such as palpitations, sweating, trembling, shortness of breath, chest pain, nausea, dizziness, derealization/depersonalization, fear of losing control or dying. The patient does not report having panic attacks.
B
Obsessive-compulsive disorder
Hint:
Obsessive-compulsive disorder (OCD) is characterized by recurrent obsessions (intrusive thoughts or images that cause anxiety) and/or compulsions (repetitive behaviors or mental acts that aim to reduce anxiety). The patient does not report having obsessions or compulsions.
C
Post-traumatic stress disorder
Hint:
Post-traumatic stress disorder (PTSD) is characterized by exposure to a traumatic event that involved actual or threatened death, serious injury, or sexual violence, and subsequent re-experiencing, avoidance, negative alterations in cognition and mood, and increased arousal related to the event. The patient denies any history of trauma.
D
Generalized anxiety disorder
E
Adjustment disorder
Hint:
Adjustment disorder is characterized by emotional or behavioral symptoms that develop within 3 months of an identifiable psychosocial stressor and cause significant impairment in social, occupational, or other areas of functioning. The symptoms usually resolve within 6 months after the termination of the stressor unless it has chronic consequences. The patient’s anxiety is not related to any specific stressors and has lasted longer than 6 months.
Question 15 Explanation: 
Generalized anxiety disorder (GAD) is characterized by persistent, excessive, and unrealistic worry about everyday things that lasts for at least 6 months. The worry causes significant distress or impairment in social, occupational, or other areas of functioning. The anxiety is not attributable to any specific triggers or stressors. The diagnosis of GAD requires at least 3 of the following symptoms: restlessness, fatigue, difficulty concentrating, irritability, muscle tension, and sleep disturbance. (Review PANRE Blueprint Topic: Generalized Anxiety Disorder)
Question 16
A 60-year-old woman with a history of smoking presents to the emergency department with acute shortness of breath and wheezing. She has been diagnosed with emphysema 5 years ago and has been using inhaled bronchodilators and corticosteroids regularly. On physical examination, she has a barrel-shaped chest, pursed-lip breathing, and decreased breath sounds bilaterally. Her arterial blood gas shows a pH of 7.35, PaCO2 of 50 mmHg, PaO2 of 60 mmHg, and HCO3- of 28 mEq/L. Which of the following mechanisms is most likely responsible for the development of emphysema in this patient?
A
Increased protease activity
B
Decreased surfactant production
Hint:
Decreased surfactant production is not a mechanism for emphysema but rather for atelectasis or respiratory distress syndrome. Surfactant is a phospholipid-protein complex that reduces surface tension at the air-liquid interface within the alveoli. Surfactant deficiency causes increased alveolar collapse especially during expiration leading to hypoxemia and hypercapnia.
C
Increased alveolar-capillary permeability
Hint:
Increased alveolar-capillary permeability is not a mechanism for emphysema but rather for acute respiratory distress syndrome (ARDS). ARDS is a condition characterized by diffuse alveolar damage due to various insults such as sepsis, trauma, or aspiration. The damage causes increased leakage of fluid, protein, and inflammatory cells into the alveolar space impairing gas exchange. ARDS presents with acute onset of dyspnea, hypoxemia, and bilateral infiltrates on chest x-ray.
D
Decreased mucociliary clearance
Hint:
Decreased mucociliary clearance is not a mechanism for emphysema but rather for chronic bronchitis which is another type of COPD. Chronic bronchitis is characterized by chronic productive cough for at least 3 months in each of 2 consecutive years. Smoking impairs mucociliary clearance by damaging cilia and increasing mucus production. This leads to mucus accumulation, bacterial colonization, and recurrent infections. Chronic bronchitis presents with cough, sputum production, wheezes, crackles, cyanosis, edema, and signs of right heart failure due to chronic hypoxia and pulmonary hypertension.
E
Increased pulmonary vascular resistance
Hint:
Increased pulmonary vascular resistance is not a mechanism for emphysema but rather a complication that may occur due to chronic hypoxia-induced vasoconstriction or destruction of pulmonary capillaries. Increased pulmonary vascular resistance may lead to pulmonary hypertension and cor pulmonale which are late manifestations of COPD. Cor pulmonale presents with right ventricular hypertrophy or dilation, jugular venous distension, hepatomegaly, ascites, and peripheral edema.
Question 16 Explanation: 
Emphysema is a type of chronic obstructive pulmonary disease (COPD) characterized by irreversible destruction of alveolar walls leading to loss of elastic recoil and air trapping. The main cause of emphysema is long-term exposure to airborne irritants such as tobacco smoke. Smoking induces inflammation and oxidative stress in the lungs which activate neutrophils and macrophages. These cells release proteases such as elastase which degrade elastin fibers that provide structural support to the alveoli. Smoking also inhibits alpha-1 antitrypsin (AAT), a protease inhibitor that normally protects the lung tissue from excessive proteolysis. The imbalance between proteases and antiproteases leads to progressive alveolar damage and enlargement, resulting in emphysema. (Review PANRE Blueprint Topic: Emphysema)
Question 17
A 65-year-old man with a history of hypertension and type 2 diabetes mellitus presents to his primary care physician for a routine follow-up visit. He has no complaints and feels well. His medications include lisinopril, metformin, and aspirin. His blood pressure is 140/80 mmHg, pulse is 72 beats per minute, and temperature is 36.5°C (97.7°F). His physical examination is unremarkable. His laboratory tests show the following:
  • Serum creatinine: 1.8 mg/dL (normal range: 0.6–1.2 mg/dL)
  • Blood urea nitrogen: 28 mg/dL (normal range: 7–20 mg/dL)
  • Urinalysis: trace proteinuria, no hematuria or casts
Based on these results, what stage of chronic kidney disease does the patient have?
A
Stage 1
Hint:
Stage 1 CKD is incorrect because it requires an eGFR of ≥90 mL/min/1.73 m^2 with evidence of kidney damage such as albuminuria or hematuria. The patient has an eGFR of approximately 34 mL/min/1.73 m^2 and trace proteinuria.
B
Stage 2
Hint:
Stage 2 CKD is incorrect because it requires an eGFR of 60–89 mL/min/1.73 m^2 with evidence of kidney damage such as albuminuria or hematuria. The patient has an eGFR of approximately 34 mL/min/1.73 m^2 and trace proteinuria.
C
Stage 3a
Hint:
Stage 3a CKD is incorrect because it requires an eGFR of 45–59 mL/min/1.73 m^2. The patient has an eGFR of approximately 34 mL/min/1.73 m^2.
D
Stage 3b
E
Stage 4
Hint:
Stage 4 CKD is incorrect because it requires an eGFR of 15–29 mL/min/1.73 m^2. The patient has an eGFR of approximately 34 mL/min/1.73 m^2.
Question 17 Explanation: 
The patient has stage 3b chronic kidney disease based on his estimated glomerular filtration rate (eGFR), which can be calculated using the Modification of Diet in Renal Disease (MDRD) equation: eGFR = [186 x serum creatinine^-1.154 x age^-0.203 x (0.742 if female)] mL/min/1.73 m^2. Plugging in the patient's values into the equation gives an eGFR of approximately 34 mL/min/1.73 m^2. Chronic kidney disease (CKD) is defined as abnormalities in kidney structure or function occurring for more than three months. CKD is classified into six stages based on eGFR: Stage | eGFR | Description
  • Stage G1 | ≥90 mL/min/1.73 m^2 | Normal or high GFR with evidence of kidney damage such as albuminuria or hematuria
  • Stage G2 |60–89 mL/min/1.73 m^2 | Mildly decreased GFR with evidence of kidney damage
  • Stage G3a |45–59 mL/min/1.73 m^2 | Mildly to moderately decreased GFR
  • Stage G3b |30–44 mL/min/1.73 m^2 | Moderately to severely decreased GFR
  • Stage G4 |15–29 mL/min/1.73 m^2 | Severely decreased GFR
  • Stage G5 |<15 mL/min/1.73 m^2 or dialysis-dependent| Kidney failure
(Review PANRE Blueprint Topic: Chronic Kidney Disease)
Question 18
A 45-year-old woman presents to the clinic with complaints of nasal congestion, facial pain, and postnasal drip for the past four months. She has tried over-the-counter antihistamines and decongestants without relief. She denies fever, cough, or sore throat. She has a history of allergic rhinitis and asthma. On physical examination, she has pale and swollen nasal mucosa with purulent discharge. There is tenderness over the maxillary sinuses bilaterally. Which of the following diagnostic tests is most appropriate for the diagnosis of chronic sinusitis in this patient?
A
CT scan of the sinuses
B
Magnetic resonance imaging (MRI) of the sinuses
Hint:
MRI of the sinuses is not the preferred initial diagnostic test for chronic sinusitis. It is reserved for cases where there is suspected intracranial extension of sinusitis or suspected complications such as abscess or osteomyelitis.
C
Plain radiography of the sinuses
Hint:
Plain radiography of the sinuses is not useful in the diagnosis of chronic sinusitis. It can only identify significant structural abnormalities and is less sensitive than CT scan.
D
Transillumination of the sinuses
Hint:
Transillumination of the sinuses is an outdated diagnostic test for sinusitis and has low diagnostic accuracy.
E
Laboratory tests for allergies
Hint:
Laboratory tests for allergies may be useful in identifying underlying allergic rhinitis that may contribute to chronic sinusitis. However, it is not the most appropriate diagnostic test for chronic sinusitis.
Question 18 Explanation: 
Chronic sinusitis is defined as inflammation of the paranasal sinuses lasting for more than 12 weeks. The diagnosis is made based on the presence of symptoms such as facial pain, nasal congestion, and nasal discharge. On physical examination, there may be tenderness over the affected sinus. CT scan of the sinuses is the most appropriate diagnostic test for chronic sinusitis. CT scan can help identify any structural abnormalities such as sinus obstruction or polyps, which can contribute to the development of chronic sinusitis. It can also help identify areas of inflammation and purulent discharge. This information can be used to guide further management. (Review PANRE Blueprint Topic: Acute and chronic sinusitis )
Question 19
A 12-year-old boy is brought to your office by his mother because of increased thirst, urination, and weight loss for the past two weeks. He also complains of fatigue and abdominal pain. His mother says he has been eating more than usual but still losing weight. His temperature is 37°C (98.6°F), blood pressure is 110/70 mm Hg, pulse is 90/min, and respirations are 20/min. Physical examination reveals dry mucous membranes, poor skin turgor, and sunken eyes. Laboratory tests show:
  • Serum glucose: 450 mg/dL
  • Serum sodium: 140 mEq/L
  • Serum potassium: 4.5 mEq/L
  • Serum bicarbonate: 12 mEq/L
  • Serum ketones: Positive
  • Urine glucose: Positive
  • Urine ketones: Positive
Which of the following is the most likely diagnosis?
A
Type 1 diabetes mellitus
Hint:
Type 1 diabetes mellitus is a chronic autoimmune disease that causes destruction of pancreatic beta cells and loss of insulin production. It typically presents in children or young adults with symptoms of hyperglycemia such as polyuria, polydipsia, polyphagia, and weight loss. However, type 1 diabetes mellitus alone does not explain the acidosis and ketosis seen in this patient.
B
Type 2 diabetes mellitus
Hint:
Type 2 diabetes mellitus is a chronic metabolic disorder that causes insulin resistance and relative insulin deficiency. It usually affects older adults who are obese or have other risk factors such as hypertension, dyslipidemia, or family history. The symptoms of hyperglycemia are similar to those of type 1 diabetes mellitus, but less severe. Type 2 diabetes mellitus rarely causes DKA unless there is severe stress or infection.
C
Diabetic ketoacidosis
D
Hyperosmolar hyperglycemic state
Hint:
Hyperosmolar hyperglycemic state (HHS) is another serious complication of type 2 diabetes mellitus that occurs when there is profound dehydration and hyperglycemia due to inadequate insulin action. Unlike DKA, HHS does not cause significant acidosis or ketosis because there is enough residual insulin to prevent lipolysis. The presentation of HHS includes extreme hyperglycemia (>600 mg/dL), hyperosmolarity (>320 mOsm/kg), dehydration, and altered mental status.
E
Maturity-onset diabetes of the young
Hint:
Maturity-onset diabetes of the young (MODY) is a rare form of monogenic diabetes that results from mutations in genes involved in beta cell function or development. It usually manifests before age 25 with mild hyperglycemia that can be controlled with oral hypoglycemic agents or diet alone. MODY does not cause DKA unless there is severe stress or infection.
Question 19 Explanation: 
Diabetic ketoacidosis (DKA) is a life-threatening complication of type 1 diabetes mellitus that occurs when there is severe insulin deficiency and increased counter-regulatory hormones (glucagon, cortisol, catecholamines, growth hormone). This leads to hyperglycemia, dehydration, electrolyte imbalance, and metabolic acidosis due to increased lipolysis and ketogenesis. The classic presentation of DKA includes polyuria, polydipsia, polyphagia, weight loss, nausea, vomiting, abdominal pain, fatigue, weakness, fruity breath odor (due to acetone), Kussmaul respirations (deep and rapid breathing to compensate for acidosis), and altered mental status. The diagnosis of DKA is based on the following criteria:
  • Hyperglycemia (>250 mg/dL)
  • Acidosis (pH <7.3 or bicarbonate <18 mEq/L)
  • Ketosis (serum or urine ketones positive)
The treatment of DKA involves fluid resuscitation with isotonic saline (0.9% NaCl), insulin infusion (regular insulin), correction of electrolyte abnormalities (especially potassium), and identification and management of precipitating factors (such as infection or noncompliance with insulin therapy). (Review PANRE Blueprint Topic: Diabetes Mellitus and Diabetic ketoacidosis (DKA))
Question 20
A 45-year-old woman with a history of asthma presents to the emergency department with fever, cough, dyspnea, and chest pain for the past three days. She reports that she did not receive the influenza vaccine this year. Her temperature is 38.9°C (102°F), blood pressure is 150/90 mmHg, pulse is 110 beats per minute, respiratory rate is 28 breaths per minute, and oxygen saturation is 92% on room air. Her physical examination reveals diffuse crackles and wheezes on lung auscultation. A chest radiograph shows bilateral patchy infiltrates. What is the most appropriate initial management of this patient?
A
Oral oseltamivir and inhaled albuterol
Hint:
Oral oseltamivir and inhaled albuterol is incorrect because it does not address the patient's hypoxemia that requires oxygen therapy. Inhaled albuterol may provide some symptomatic relief for bronchospasm but does not treat the underlying viral infection.
B
Intravenous ceftriaxone and azithromycin
Hint:
Intravenous ceftriaxone and azithromycin is incorrect because it is an empirical antibiotic regimen for bacterial pneumonia but not for viral pneumonia caused by influenza viruses. Antibiotics are not indicated unless there is evidence of a secondary bacterial infection such as persistent or worsening fever, leukocytosis, purulent sputum, or cavitation on chest radiograph.
C
Intravenous methylprednisolone and salbutamol
Hint:
Intravenous methylprednisolone and salbutamol is incorrect because it is a treatment for acute exacerbation of asthma but not for influenza pneumonia. Corticosteroids may worsen viral replication and increase the risk of complications in patients with influenza pneumonia. Salbutamol (or albuterol) may provide some symptomatic relief for bronchospasm but does not treat the underlying viral infection.
D
Intravenous oseltamivir and oxygen therapy
E
Oral amoxicillin-clavulanate and montelukast
Hint:
Oral amoxicillin-clavulanate and montelukast is incorrect because it is a treatment for chronic asthma control but not for influenza pneumonia. Amoxicillin-clavulanate is an antibiotic that has no activity against influenza viruses. Montelukast is a leukotriene receptor antagonist that reduces inflammation in asthma but has no effect on viral infections. Neither drug addresses the patient's hypoxemia which requires oxygen therapy.
Question 20 Explanation: 
The patient has signs and symptoms of influenza pneumonia, which is a viral infection of the lower respiratory tract caused by influenza A or B viruses. Influenza pneumonia can cause severe hypoxemia, respiratory failure, and death in high-risk groups such as patients with chronic lung diseases like asthma. The most appropriate initial management of influenza pneumonia is antiviral therapy with oseltamivir (or another neuraminidase inhibitor) within 48 hours of symptom onset to reduce viral shedding, duration of symptoms, complications, and mortality. Oseltamivir can be given orally or intravenously depending on the patient's condition. Oxygen therapy should also be provided to maintain adequate oxygenation. (Review PANRE Blueprint Topic: Viral pneumonia)
Question 21
A 60-year-old male presents to the emergency department with chest pain and shortness of breath. ECG reveals ST segment elevation in the anterior leads. Which of the following laboratory tests is the most specific for the diagnosis of acute myocardial infarction (AMI)?
A
Troponin
B
Creatinine kinase-MB (CK-MB)
Hint:
Creatinine kinase (CK) is an enzyme found in various tissues, including skeletal and cardiac muscle. CK-MB is a specific isoform of CK found predominantly in cardiac muscle cells. Elevated levels of CK-MB can be seen in the early stages of AMI, but CK-MB is less specific for the diagnosis of AMI compared to troponin.
C
Myoglobin
Hint:
Myoglobin is a protein found in skeletal and cardiac muscle cells. Elevated myoglobin levels can be seen within 1-3 hours after the onset of symptoms, but myoglobin is less specific for the diagnosis of AMI compared to troponin. Elevated myoglobin levels can also be seen in other conditions that cause muscle injury, such as rhabdomyolysis.
D
C-reactive protein (CRP)
Hint:
C-reactive protein (CRP) is an acute-phase protein that is elevated in response to tissue injury, inflammation, and infection. While elevated CRP levels can be seen in patients with AMI, CRP is not specific for the diagnosis of AMI and cannot be used as a diagnostic tool on its own.
E
Brain natriuretic peptide (BNP)
Hint:
Brain natriuretic peptide (BNP) is a hormone released by the heart in response to increased pressure and volume in the cardiac chambers. Elevated BNP levels can be seen in patients with heart failure and other cardiac conditions, but BNP is not specific for the diagnosis of AMI.
Question 21 Explanation: 
Acute myocardial infarction (AMI) is a serious medical emergency that requires prompt diagnosis and treatment. Cardiac biomarkers such as troponin, creatinine kinase-MB (CK-MB), and myoglobin are commonly used to diagnose AMI. Among these biomarkers, troponin is the most specific for the diagnosis of AMI. Troponin is a protein found in cardiac muscle cells, and its release into the bloodstream is a specific marker of myocardial injury. Elevated troponin levels are typically seen within 3-4 hours after the onset of symptoms and can remain elevated for up to 10-14 days after an AMI. (Review PANRE Blueprint Topic: Acute myocardial infarction: ST-segment elevation myocardial infarction (STEMI))
Question 22

A 35-year-old man visits his allergist for evaluation of his chronic nasal symptoms. He says that he has been suffering from sneezing, runny nose, and nasal congestion for more than six months. He also complains of frequent headaches and sinus infections. He has tried various over-the-counter medications but none of them have provided adequate relief. He has a history of asthma and eczema since childhood. On physical examination, he has dark circles under his eyes, a transverse nasal crease, and pale boggy nasal mucosa with clear discharge. Which of the following is the most appropriate initial treatment for this patient?

A
Intranasal corticosteroids
B
Oral antihistamines
Hint:
Oral antihistamines are effective for mild to moderate allergic rhinitis. They block histamine receptors and reduce symptoms such as sneezing, rhinorrhea, and itching. However, they have less effect on nasal congestion than intranasal corticosteroids. First-generation antihistamines may also cause sedation or anticholinergic side effects.
C
Decongestant nasal sprays
Hint:
Decongestant nasal sprays are useful for short-term relief of nasal congestion due to allergic rhinitis. They act by constricting blood vessels in the nasal mucosa and reducing edema. However, they should not be used for more than three to five days because they can cause rebound congestion (rhinitis medicamentosa).
D
Leukotriene receptor antagonists
Hint:
Leukotriene receptor antagonists are oral medications that block leukotriene receptors and reduce inflammation in allergic rhinitis. They may be used as an alternative or adjunctive therapy to intranasal corticosteroids or antihistamines. However, they are less effective than intranasal corticosteroids as monotherapy for allergic rhinitis.
E
Immunotherapy
Hint:
Immunotherapy is a long-term treatment that involves exposing the patient to gradually increasing doses of allergens to induce tolerance and reduce symptoms. It may be considered for patients with severe allergic rhinitis who do not respond well to pharmacotherapy or who have significant adverse effects from medications. However, it is not an initial treatment option because it requires several months to years to achieve optimal results.
Question 22 Explanation: 
Intranasal corticosteroids are the first-line treatment for moderate to severe allergic rhinitis because they are the most effective treatment and have few systemic effects. They reduce inflammation and edema of the nasal mucosa and improve symptoms such as sneezing, rhinorrhea, congestion, and itching. They also have beneficial effects on comorbid conditions such as asthma, sinusitis, and otitis media. (Review PANRE Blueprint Topic: Rhinitis)
Question 23
A 65-year-old man presents to your office with complaints of constipation for the past six months. He says that he has difficulty passing stools, which are hard and dry. He also reports occasional abdominal pain and bloating. He denies any weight loss, blood in stools, fever, or night sweats. His medical history is significant for hypertension and type 2 diabetes mellitus. His medications include metformin, lisinopril, and aspirin. He does not smoke or drink alcohol. On physical examination, his vital signs are normal. His abdomen is soft and nontender, with normal bowel sounds. There are no masses or organomegaly palpable. Which of the following is the most appropriate next step in evaluating this patient?
A
Colonoscopy
B
Barium enema
Hint:
Barium enema is an imaging test that involves injecting a contrast agent (barium sulfate) into the rectum and taking X-rays of the colon. It can show structural abnormalities such as diverticula, strictures, masses, or volvulus. However, it is less sensitive and specific than colonoscopy for detecting colorectal malignancy. Therefore, it is not the preferred test for this patient.
C
Thyroid function tests
Hint:
Thyroid function tests are blood tests that measure the levels of thyroid hormones (T3, T4) and thyroid-stimulating hormone (TSH). They can help diagnose thyroid disorders such as hypothyroidism or hyperthyroidism. Hypothyroidism can cause constipation due to decreased gastrointestinal motility. However, this patient has no other signs or symptoms of hypothyroidism such as fatigue, cold intolerance, dry skin, hair loss, or bradycardia. Therefore, thyroid function tests are not necessary for this patient.
D
Stool osmolarity
Hint:
Stool osmolarity is a test that measures the concentration of solutes in stool water. It can help differentiate between osmotic diarrhea (high stool osmolarity) and secretory diarrhea (low stool osmolarity). However, this test is not useful for evaluating constipation which is defined by infrequent or difficult defecation.
E
Dietary modification
Hint:
Dietary modification is a nonpharmacological measure that involves increasing fiber and fluid intake to improve stool consistency and frequency. It may be effective for patients with primary constipation which has no identifiable organic cause. However, this patient has chronic constipation which requires further evaluation before initiating treatment.
Question 23 Explanation: 
Colonoscopy is a procedure that involves inserting a flexible tube with a camera into the colon to visualize the mucosa and detect any abnormalities such as polyps, tumors, inflammation, or bleeding. It is indicated for patients with chronic constipation who are older than 50 years or have any red flag features for colorectal malignancy such as weight loss, blood in stools, anemia, or family history of colon cancer. This patient meets the age criterion and should undergo colonoscopy to rule out any serious causes of his constipation. (Review PANRE Blueprint Topic: Constipation)
Question 24
A 7-year-old girl is brought to the clinic by her mother because of fever, sore throat, and difficulty swallowing for the past three days. She also has a rash on her trunk and extremities that started yesterday. She has no history of allergies or chronic medical conditions. Her temperature is 38.5°C (101.3°F), blood pressure is 100/60 mm Hg, pulse is 110/min, and respirations are 24/min. Physical examination reveals erythema and exudates on the tonsils, tender anterior cervical lymph nodes, and a blanching erythematous rash with sandpaper-like texture. A rapid antigen test for group A streptococcus (GAS) is positive. Which of the following is the most appropriate next step in management?
A
Administer intramuscular penicillin G benzathine
B
Administer oral amoxicillin-clavulanate
Hint:
Oral amoxicillin-clavulanate is an alternative antibiotic for patients with GAS pharyngitis who are allergic to penicillin. However, it has more side effects (such as diarrhea or rash) than penicillin and does not offer any advantage over penicillin in terms of efficacy or spectrum
C
Administer oral azithromycin
Hint:
Oral azithromycin is another alternative antibiotic for patients with GAS pharyngitis who are allergic to penicillin. However, it has less activity against GAS than penicillin and may increase the risk of macrolide resistance
D
Order a throat culture for confirmation
Hint:
Ordering a throat culture for confirmation is not necessary because the rapid antigen test for GAS has high specificity (>95%) and can reliably diagnose GAS pharyngitis without further testing. Throat culture may be indicated if the rapid antigen test is negative, but there is high clinical suspicion for GAS infection.
E
Order an antistreptolysin O (ASO) titer
Hint:
Ordering an antistreptolysin O (ASO) titer is not useful for diagnosing acute GAS pharyngitis because it measures antibodies that develop after several weeks of infection. ASO titer may be helpful for confirming past exposure to GAS in patients with suspected rheumatic fever or glomerulonephritis.
Question 24 Explanation: 
The patient has scarlet fever, a complication of GAS pharyngitis that causes a diffuse erythematous rash that typically spares the palms and soles. The diagnosis of scarlet fever is based on clinical features and a positive rapid antigen test or throat culture for GAS. The treatment of choice for GAS pharyngitis and scarlet fever is penicillin. Intramuscular penicillin G benzathine is preferred over oral penicillin V because it provides better compliance and efficacy. The treatment reduces the risk of suppurative complications (such as peritonsillar abscess or otitis media) and nonsuppurative complications (such as rheumatic fever or glomerulonephritis). (Review PANRE Blueprint Topic: Acute Pharyngitis)
Question 25
A 32-year-old man with a history of intravenous drug use presents to the emergency department with fever, chills, and severe pain in his right groin for the past two days. He denies any trauma, sexual activity, or recent travel. On physical examination, he has a large, erythematous, fluctuant mass in his right inguinal area with surrounding cellulitis and lymphangitis. His vital signs are: temperature 39°C (102.2°F), blood pressure 150/90 mmHg, pulse 110 beats/min, respirations 20 breaths/min, and oxygen saturation 98% on room air. His white blood cell count is 18,000/mm3 with 80% neutrophils. Which of the following is the most likely causative organism of this patient’s infection?
A
Escherichia coli
Hint:
Escherichia coli is a gram-negative bacillus that can cause urinary tract infections (UTIs), intra-abdominal infections, septicemia, and meningitis. It is not a common cause of SSTIs.
B
Streptococcus pyogenes
Hint:
Streptococcus pyogenes can also cause cellulitis, erysipelas, necrotizing fasciitis, and toxic shock syndrome. It is a possible cause of SSTIs but less likely than S. aureus in this patient.
C
Staphylococcus aureus
D
Pseudomonas aeruginosa
Hint:
Pseudomonas aeruginosa is a gram-negative bacillus that can cause pneumonia, UTIs, septicemia, and infections in immunocompromised patients. It can also cause SSTIs in patients with burns or wounds exposed to contaminated water. It is not a likely cause of this patient’s infection.
E
Clostridium perfringens
Hint:
Clostridium perfringens is a gram-positive bacillus that can cause gas gangrene, food poisoning, and enterocolitis. It can cause SSTIs in patients with trauma or surgery involving devitalized tissue. It is not a likely cause of this patient’s infection.
Question 25 Explanation: 
This patient has an abscess in his right groin due to an infection of the skin and soft tissue. The most common cause of skin and soft tissue infections (SSTIs) in the United States is Staphylococcus aureus, especially methicillin-resistant S. aureus (MRSA). MRSA is more prevalent among patients with risk factors such as intravenous drug use, which this patient has. (Review PANRE Blueprint Topic: Cellulitis )
Question 26
Which of the following organisms is the most common causative agent of erysipelas?
A
Streptococcus pneumoniae
Hint:
Streptococcus pneumoniae is a common cause of pneumonia and meningitis, but it is not a frequent cause of erysipelas.
B
Staphylococcus aureus
Hint:
Staphylococcus aureus is a common cause of skin and soft tissue infections, but it is less common than S. pyogenes in causing erysipelas.
C
Pseudomonas aeruginosa
Hint:
Pseudomonas aeruginosa is a gram-negative bacillus that is associated with infections in immunocompromised patients, but it is not a typical cause of erysipelas.
D
Streptococcus pyogenes
E
Haemophilus influenzae
Hint:
Haemophilus influenzae is a gram-negative coccobacillus that can cause respiratory tract infections, but it is not a common cause of erysipelas.
Question 26 Explanation: 
Erysipelas is a bacterial infection of the skin and subcutaneous tissue that is caused by Streptococcus pyogenes, also known as group A streptococcus. This organism is responsible for approximately 80-90% of cases of erysipelas. The infection usually starts as a small, red, raised area of skin that spreads rapidly and causes pain, fever, and systemic symptoms. Other common causative agents of erysipelas include Staphylococcus aureus and Streptococcus pneumoniae, but they are less common than S. pyogenes. (Review PANRE Blueprint Topic: Erysipelas )
Question 27
A 3-year-old boy is brought to your by his mother because of fever and ear pain for the past two days. He has had three episodes of otitis media in the past six months. His temperature is 38.5°C (101.3°F), pulse is 110/min, respirations are 22/min, and blood pressure is 100/60 mm Hg. Physical examination shows a red and bulging tympanic membrane in the right ear with decreased mobility on pneumatic otoscopy. The left ear is normal. There is no nasal discharge or lymphadenopathy. Which of the following is the most appropriate next step in management?
A
Amoxicillin-clavulanate
B
Azithromycin
Hint:
Azithromycin is incorrect because it has poor activity against S. pneumoniae, which accounts for about 30% of AOM cases.
C
Ceftriaxone
Hint:
Ceftriaxone is incorrect because it is a parenteral antibiotic that should be reserved for patients who cannot tolerate oral antibiotics or have failed initial therapy.
D
Myringotomy
Hint:
Myringotomy is incorrect because it is a surgical procedure that involves making an incision in the tympanic membrane to drain fluid from the middle ear. It is indicated for patients with chronic otitis media with effusion (OME), which is a persistent accumulation of fluid in the middle ear without signs of infection.
E
Observation
Hint:
Observation is incorrect because it may be an option for selected patients older than 2 years with mild symptoms and no risk factors for complications, but not for this patient who has severe symptoms and recurrent infections.
Question 27 Explanation: 
The patient has acute otitis media (AOM), which is an infection of the middle ear caused by bacteria or viruses. The most common bacterial pathogens are Streptococcus pneumoniae, Haemophilus influenzae, and Moraxella catarrhalis. The mainstay of treatment for AOM is antibiotics, especially in children younger than 2 years or those with severe symptoms or recurrent infections. Amoxicillin-clavulanate is the first-line antibiotic for AOM because it covers all three common pathogens and has activity against beta-lactamase-producing strains of H. influenzae and M. catarrhalis. (Review PANRE Blueprint Topic: Acute Otitis Media)
Question 28
Which of the following is a key diagnostic criterion for type 1 diabetes mellitus?
A
Age of onset over 40 years old
Hint:
Type 2 diabetes is more commonly seen in individuals over 40 years old.
B
Elevated C-peptide levels
Hint:
Elevated C-peptide levels indicate the presence of endogenous insulin production, which is more commonly seen in type 2 diabetes.
C
Positive islet cell antibodies
D
Obesity
Hint:
While obesity can be a risk factor for type 2 diabetes, it is not a key diagnostic criterion for type 1 diabetes.
E
Family history of type 2 diabetes
Hint:
While family history can be a risk factor for type 2 diabetes, it is not a key diagnostic criterion for type 1 diabetes.
Question 28 Explanation: 
Type 1 diabetes mellitus is characterized by the autoimmune destruction of pancreatic beta cells, resulting in insulin deficiency. The presence of islet cell antibodies is a key diagnostic criterion for type 1 diabetes. Elevated C-peptide levels indicate the presence of endogenous insulin production and are typically seen in type 2 diabetes. Age of onset over 40 years old is more commonly seen in type 2 diabetes. Obesity and a family history of type 2 diabetes are also more commonly seen in type 2 diabetes. (Review PANRE Blueprint Topic: Diabetes mellitus type 1 )
Question 29
A 35-year-old woman presents to your office with a 10-day history of nasal congestion, purulent nasal discharge, facial pain, and headache. She says she initially had a sore throat and low-grade fever but those symptoms resolved after a few days. She has tried over-the-counter decongestants and saline irrigation but has not noticed any improvement. She has no known drug allergies. On physical examination, she has tenderness over her left maxillary sinus and erythema of the nasal mucosa. A CT scan of the sinuses shows opacification of the left maxillary sinus. Which of the following is the most appropriate next step in management?
A
Observation for another 7 days
Hint:
Observation for another 7 days is inappropriate because the patient already has persistent symptoms (>10 days) that indicate bacterial infection rather than viral infection. Viral sinusitis usually resolves within 7-10 days with supportive therapy such as analgesia, decongestants, hydration, and saline irrigation.
B
Amoxicillin-clavulanate for 10-14 days
C
Levofloxacin for 10-14 days
Hint:
Levofloxacin for 10-14 days is an alternative option for patients with penicillin allergy but not the first-line choice because of its broader spectrum of activity and potential adverse effects, such as tendon rupture and QT prolongation.
D
Intranasal corticosteroids for 4 weeks
Hint:
Intranasal corticosteroids for 4 weeks (choice D) are not recommended as monotherapy for acute bacterial sinusitis because they do not have antibacterial activity and may delay resolution of infection. They may be used as adjunctive therapy to reduce inflammation and improve drainage in patients with chronic or recurrent sinusitis.
E
Endoscopic sinus surgery
Hint:
Endoscopic sinus surgery is reserved for patients with complications of acute bacterial sinusitis such as orbital cellulitis, intracranial abscess, or osteomyelitis; or patients with chronic or recurrent sinusitis that fails medical therapy.
Question 29 Explanation: 
The patient has acute bacterial sinusitis, which is defined as an infection of the paranasal sinuses lasting less than 4 weeks with complete resolution. The most common causative organisms are Streptococcus pneumoniae, Haemophilus influenzae, and Moraxella catarrhalis. The diagnosis is based on clinical criteria such as persistent symptoms (>10 days), severe symptoms (high fever, purulent discharge), or worsening symptoms after initial improvement. A CT scan can confirm the diagnosis but is not routinely indicated unless there are complications or treatment failure. The first-line treatment for acute bacterial sinusitis is amoxicillin-clavulanate for 10-14 days. This antibiotic covers both gram-positive and gram-negative bacteria and has activity against beta-lactamase-producing strains. If the patient has a penicillin allergy, alternative options include doxycycline or respiratory fluoroquinolones (e.g., levofloxacin). (Review PANRE Blueprint Topic: Acute and Chronic Sinusitis)
Question 30

A 65-year-old man with a history of hypertension and peptic ulcer disease presents to your clinic for a routine follow-up. He reports feeling well and has no complaints. His medications include lisinopril and omeprazole. His vital signs are normal. A complete blood count (CBC) shows:

  • Hemoglobin: 10 g/dL (normal: 13-17 g/dL)
  • Hematocrit: 30% (normal: 40-50%)
  • Mean corpuscular volume (MCV): 70 fL (normal: 80-100 fL)
  • Red cell distribution width (RDW): 18% (normal: 11-15%)
  • White blood cell count: 6 x 10^9/L (normal: 4-11 x 10^9/L)
  • Platelet count: 250 x 10^9/L (normal: 150-450 x 10^9/L)
What is the most likely cause of this patient’s anemia?
A
Chronic kidney disease
Hint:
Chronic kidney disease can cause normocytic anemia due to reduced production of erythropoietin by the kidneys. The MCV would be normal (80-100 fL).
B
Folate deficiency
Hint:
Folate deficiency can cause macrocytic anemia due to impaired DNA synthesis in red blood cell precursors. The MCV would be high (>100 fL).
C
Gastrointestinal bleeding
D
Thalassemia trait
Hint:
Thalassemia trait can cause microcytic anemia due to reduced synthesis of alpha or beta globin chains that form hemoglobin. However, thalassemia trait usually has a normal or low RDW (<15%) because red blood cells are uniformly small. Thalassemia trait also has a genetic basis and is more common in people of Mediterranean, African, or Southeast Asian descent.
E
Vitamin B12 deficiency
Hint:
Vitamin B12 deficiency can also cause macrocytic anemia due to impaired DNA synthesis in red blood cell precursors as well as neurological symptoms such as peripheral neuropathy, ataxia, dementia, or psychosis. The MCV would be high (>100 fL).
Question 30 Explanation: 
The patient has a microcytic anemia, which is characterized by a low MCV (<80 fL). The most common cause of microcytic anemia is iron deficiency. Iron deficiency can result from inadequate dietary intake, malabsorption, increased demand, or chronic blood loss. In this patient, the most likely source of chronic blood loss is gastrointestinal bleeding due to his history of peptic ulcer disease and the use of omeprazole. Omeprazole can mask the symptoms of gastrointestinal bleeding by reducing acid secretion and healing ulcers but does not prevent recurrence or complications. The patient may also have occult blood loss that is not visible in the stool. The RDW is elevated (>15%), indicating increased variation in red blood cell size due to iron deficiency. To confirm iron deficiency, iron studies such as serum ferritin, serum iron, total iron-binding capacity (TIBC), and transferrin saturation should be ordered. The patient should also undergo endoscopy to evaluate for the source and severity of gastrointestinal bleeding and rule out malignancy. The other answer choices are incorrect because they are not causes of microcytic anemia but rather causes of normocytic or macrocytic anemia. (Review PANRE Blueprint Topic: Iron Deficiency Anemia)
Question 31
A 35-year-old woman presents to the emergency department with palpitations, chest pain, shortness of breath, and fear of dying. She says this is the third time this month that she has experienced these symptoms without any apparent trigger. She has no history of cardiac or pulmonary disease and does not smoke or drink alcohol. Her vital signs are normal and her physical examination is unremarkable. An electrocardiogram shows sinus tachycardia but no ischemic changes. A chest radiograph is normal. Which of the following is the most likely diagnosis?
A
Acute coronary syndrome
Hint:
Acute coronary syndrome is unlikely in this patient given her young age, lack of cardiac risk factors, normal vital signs and electrocardiogram.
B
Asthma
Hint:
Asthma can cause shortness of breath and chest tightness, but it is usually associated with wheezes on auscultation and triggered by allergens or irritants.
C
Panic disorder
D
Pulmonary embolism
Hint:
Pulmonary embolism can present with sudden onset dyspnea and chest pain, but it often causes hypoxia, tachypnea, and hypotension as well as abnormal findings on electrocardiogram (e.g., S1Q3T3 pattern) or chest radiograph (e.g., Hampton’s hump).
E
Thyrotoxicosis
Hint:
Thyrotoxicosis can cause palpitations and anxiety, but it also typically causes weight loss, heat intolerance, tremor, and exophthalmos, as well as abnormal thyroid function tests.
Question 31 Explanation: 
Panic disorder is characterized by recurrent and unexpected panic attacks that cause significant distress or impairment in functioning. A panic attack is a sudden episode of intense fear or discomfort that peaks within minutes and is accompanied by at least four of the following symptoms: palpitations, sweating, trembling, shortness of breath, chest pain, nausea, dizziness, chills or heat sensations, paresthesias, derealization or depersonalization, fear of losing control or going crazy, and fear of dying. The diagnosis of panic disorder requires at least one of the attacks to be followed by one month or more of persistent worry about having another attack or its consequences (e.g., having a heart attack), or a significant change in behavior related to the attacks (e.g., avoiding situations that might trigger them). The patient in this scenario meets these criteria and has no evidence of any other medical condition that could explain her symptoms. (Review PANRE Blueprint Topic: Panic Disorder)
Question 32
A 55-year-old man with a history of hypertension, diabetes, and smoking presents to his primary care provider for a routine check-up. He denies any chest pain, dyspnea, or palpitations. His blood pressure is 160/90 mmHg, pulse is 80 beats/min, and respiratory rate is 16 breaths/min. His physical examination is unremarkable. His fasting lipid profile shows a total cholesterol of 250 mg/dL, HDL of 35 mg/dL, LDL of 180 mg/dL, and triglycerides of 200 mg/dL. His fasting blood glucose is 150 mg/dL. What is the most appropriate next step in the management of this patient?
A
Refer him for coronary angiography
Hint:
This is an invasive procedure that involves inserting a catheter into the coronary arteries to visualize any blockages or narrowing. It is not indicated for asymptomatic patients with coronary artery disease (CAD), unless they have high-risk features such as diabetes with renal impairment, left main or multivessel disease, or reduced left ventricular function.
B
Prescribe him a statin and an aspirin
C
Advise him to quit smoking and exercise regularly
Hint:
This is an important part of lifestyle modification for patients with CAD, but it is not sufficient by itself. Smoking cessation reduces the risk of myocardial infarction (MI), stroke, and death by about 50% within one year. Physical activity improves cardiovascular fitness, lowers blood pressure, enhances insulin sensitivity, and reduces inflammation. However, these interventions should be combined with pharmacological therapy for optimal outcomes.
D
Order an electrocardiogram (ECG) and an echocardiogram
Hint:
These are non-invasive tests that can assess the electrical activity and structural function of the heart respectively. They are useful for diagnosing CAD in symptomatic patients or those with abnormal findings on physical examination. However, they are not routinely indicated for screening asymptomatic patients with CAD unless they have specific indications such as family history of sudden cardiac death or arrhythmias. Echocardiogram may also be helpful to evaluate left ventricular function before starting beta-blockers.
E
Start him on a beta-blocker and an ACE inhibitor
Hint:
These are antihypertensive drugs that can lower blood pressure and reduce cardiac workload in patients with CAD. Beta-blockers block beta-adrenergic receptors in the heart and reduce heart rate, contractility, and oxygen demand ACE inhibitors inhibit angiotensin-converting enzyme (ACE), which converts angiotensin I to angiotensin II, a potent vasoconstrictor that also stimulates aldosterone secretion. Aldosterone increases sodium reabsorption, water retention, and blood volume. Both beta-blockers and ACE inhibitors have been shown to improve survival and prevent MI in patients with CAD who have reduced left ventricular function or previous MI[ However], they are not first-line agents for primary prevention of CAD in asymptomatic patients without these conditions.
Question 32 Explanation: 
Statins are cholesterol-lowering drugs that reduce the risk of cardiovascular events in patients with CAD by stabilizing plaques and improving endothelial function. Aspirin is an antiplatelet agent that prevents thrombosis by inhibiting cyclooxygenase-1 (COX-1) enzyme in platelets. Both statins and aspirin are recommended for primary prevention of CAD in patients with high-risk features such as diabetes, hypertension, smoking, or dyslipidemia. (Review PANRE Blueprint Topic: Coronary artery disease)
Question 33

A 32-year-old female with a history of recurrent episodes of cellulitis presents with erythema and edema of her left lower leg. She has been treated for cellulitis multiple times in the past with antibiotics. On examination, the area is tender, warm to touch, and has small vesicles on the surface. Gram stain and culture of the vesicular fluid are negative. What is the most likely cause of this patient's cellulitis?

A
Group A streptococcus
Hint:
Group A streptococcus and B) Staphylococcus aureus are common bacterial causes of cellulitis, but the presence of vesicles and negative culture makes these less likely.
B
Staphylococcus aureus
Hint:
Group A streptococcus and B) Staphylococcus aureus are common bacterial causes of cellulitis, but the presence of vesicles and negative culture makes these less likely.
C
Mycobacterium marinum
D
Pseudomonas aeruginosa
Hint:
Pseudomonas aeruginosa is a common cause of skin infections in individuals who have been in contact with contaminated water, but it typically causes more severe infections and is associated with necrotizing fasciitis rather than cellulitis.
E
Haemophilus influenzae
Hint:
Haemophilus influenzae is a rare cause of cellulitis and is typically associated with underlying immunodeficiency or malignancy.
Question 33 Explanation: 
The presentation of erythema, edema, and vesicles is suggestive of atypical or non-bacterial cellulitis, which can be caused by Mycobacterium marinum, a slow-growing bacterium that is commonly found in saltwater environments. It can cause skin infections in individuals who have come into contact with contaminated water, such as fish handlers or aquarium workers. Gram stain and culture of the vesicular fluid may be negative, and diagnosis is typically made by biopsy and culture of tissue samples. Treatment typically involves a combination of antibiotics, such as rifampin and clarithromycin, and surgical debridement of infected tissue. (Review PANRE Blueprint Topic: Cellulitis)
Question 34
A 65-year-old man presents to the emergency department with sudden onset of left-sided weakness and slurred speech that lasted for 15 minutes and resolved completely. He has a history of hypertension, diabetes mellitus, and hyperlipidemia. He takes lisinopril, metformin, and atorvastatin. His vital signs are blood pressure 180/100 mmHg, pulse 90 beats/min, respiratory rate 18 breaths/min, temperature 36.5°C (97.7°F), and oxygen saturation 98% on room air. His physical examination is normal except for mild left-sided facial droop. His electrocardiogram shows normal sinus rhythm without ischemic changes. A noncontrast head CT scan is negative for acute hemorrhage or infarction. What is the most appropriate next step in management?
A
Admit to intensive care unit for intravenous thrombolysis
Hint:
Intravenous thrombolysis with tissue plasminogen activator (tPA) is indicated for patients with acute ischemic stroke who present within 4.5 hours of symptom onset and have no contraindications. However, this patient does not have an acute ischemic stroke because his symptoms resolved completely within an hour and his head CT scan was negative for infarction. Therefore, he does not qualify for intravenous thrombolysis.
B
Admit to stroke unit for aspirin and clopidogrel
C
Discharge home with aspirin and outpatient evaluation
Hint:
Discharging this patient home with aspirin only would be inappropriate because he has a moderate risk of developing a stroke within the next few days. He needs more aggressive antiplatelet therapy with aspirin and clopidogrel as well as close monitoring in a hospital setting.
D
Perform carotid ultrasound and magnetic resonance angiography
Hint:
Performing carotid ultrasound and magnetic resonance angiography may be useful to evaluate the source of embolism in this patient but they are not urgent tests that need to be done immediately. They can be performed after initiating antiplatelet therapy and stabilizing the patient’s blood pressure. Furthermore, these tests do not alter the initial management strategy which is based on clinical criteria rather than imaging findings.
E
Perform echocardiogram and Holter monitor
Hint:
Performing echocardiogram and Holter monitor may also be helpful to identify potential cardiac sources of embolism such as atrial fibrillation or valvular disease in this patient but they are not essential tests that need to be done right away. They can be done after starting antiplatelet therapy and controlling the patient’s blood pressure. Moreover, these tests do not change the initial management plan which is based on clinical criteria rather than imaging results.
Question 34 Explanation: 
This patient has a transient ischemic attack (TIA), which is defined as a brief episode of focal neurologic dysfunction caused by ischemia without evidence of acute infarction. TIAs are considered a medical emergency because they are associated with a high risk of subsequent stroke. The ABCD2 score is a risk assessment tool that determines the risk of a stroke shortly after a TIA. The score ranges from 0 to 7 based on age, blood pressure, clinical features, duration of symptoms, and presence of diabetes. This patient has an ABCD2 score of 4 (1 point each for age >60 years, blood pressure >140/90 mmHg, unilateral weakness, and duration >10 minutes), which indicates a moderate risk of stroke within 2 days (9.8%). The most appropriate next step in management for this patient is to admit him to a stroke unit for antiplatelet therapy with aspirin and clopidogrel. This combination has been shown to reduce the risk of recurrent stroke compared to aspirin alone in patients with recent TIA or minor ischemic stroke. The benefit is greatest when initiated within 24 hours of symptom onset. (Review PANRE Blueprint Topic: Transient Ischemic Attack)
Question 35
A 35-year-old male presents to the clinic with a BMI of 32 kg/m2 and complains of shortness of breath and fatigue. He reports a sedentary lifestyle and a diet high in processed foods. Which of the following is the most likely comorbidity associated with this patient's obesity?
A
Osteoporosis
Hint:
Osteoporosis is not typically associated with obesity, but rather with conditions such as hyperthyroidism, malabsorption syndromes, and chronic use of corticosteroids.
B
Gout
Hint:
Gout is associated with hyperuricemia, which can be exacerbated by obesity, but it is not a direct consequence of obesity itself.
C
Chronic obstructive pulmonary disease (COPD)
Hint:
Chronic obstructive pulmonary disease (COPD) is typically associated with smoking and environmental exposures, and while obesity can contribute to COPD, it is not the most likely comorbidity in this case.
D
Non-alcoholic fatty liver disease (NAFLD)
E
Iron-deficiency anemia
Hint:
Iron-deficiency anemia can be associated with chronic gastrointestinal bleeding, which is not typically associated with obesity.
Question 35 Explanation: 
Obesity is associated with numerous comorbidities, including type 2 diabetes, hypertension, sleep apnea, cardiovascular disease, and NAFLD. The patient's complaints of shortness of breath and fatigue are likely related to obesity-related complications, such as sleep apnea and reduced physical activity. NAFLD is the most common liver disease associated with obesity, and it can progress to non-alcoholic steatohepatitis (NASH) and liver cirrhosis if left untreated. (Review PANRE Blueprint Topic: Obesity)
Question 36

A 65-year-old man presents to his primary care provider with complaints of intermittent claudication in his left calf. He says that he feels pain and cramping in his leg after walking for about 10 minutes, which is relieved by resting. He has a history of hypertension, hyperlipidemia, and smoking. On physical examination, his left lower extremity pulses are diminished compared to the right. His ankle-brachial index (ABI) is 0.7 on the left and 1.0 on the right. What is the most likely cause of his symptoms?

A
Atherosclerosis
B
Vasculitis
Hint:
Vasculitis is an inflammation of blood vessels that can cause narrowing or occlusion of arteries. However, vasculitis usually affects younger patients and causes other systemic symptoms such as fever, weight loss, rash, joint pain, etc. Vasculitis can also affect multiple organs such as kidneys, lungs, eyes etc.
C
Trauma
Hint:
Trauma can cause injury to blood vessels that may result in reduced blood flow to an extremity. However, trauma usually causes acute onset of symptoms such as pain, swelling, bruising, bleeding, etc. Trauma can also be associated with fractures, dislocations, nerve damage etc.
D
Radiation exposure
Hint:
Radiation exposure can cause damage to blood vessels that may lead to stenosis or occlusion over time. However, radiation exposure usually affects patients who have received radiation therapy for cancer or other conditions. Radiation exposure can also cause other complications such as skin changes, fibrosis, necrosis etc.
E
Muscle strain
Hint:
Muscle strain is an injury to muscle fibers that causes pain and stiffness in the affected area. However, muscle strain usually occurs after excessive or unusual physical activity such as lifting heavy objects or sports. Muscle strain can also be associated with swelling, redness, warmth, etc.
Question 36 Explanation: 
Peripheral artery disease (PAD) is a common condition in which narrowed arteries reduce blood flow to the arms or legs. The most common cause of PAD is atherosclerosis, which is a buildup of fatty, cholesterol-containing deposits (plaques) on artery walls. PAD typically affects older adults with risk factors such as hypertension, hyperlipidemia, smoking, diabetes, and obesity. Patients with PAD may present with intermittent claudication, which is pain and cramping in the affected limb triggered by exercise and relieved by rest. The diagnosis of PAD can be confirmed by measuring the ABI, which is the ratio of systolic blood pressure at the ankle to that at the brachial artery. An ABI less than 0.9 indicates PAD.
Question 37
Which of the following is an appropriate step-down therapy for a patient with well-controlled asthma on medium-dose inhaled corticosteroids (ICS) and long-acting beta agonists (LABA)?
A
Discontinue LABA and continue medium-dose ICS
Hint:
Discontinuing LABA and continuing medium-dose ICS is not an appropriate step-down option for this patient. This would correspond to step 3 of the asthma treatment algorithm which still involves a relatively high dose of ICS. Moreover, discontinuing LABA abruptly may increase the risk of exacerbations or loss of control due to withdrawal effects. Therefore, if LABA is discontinued, it should be done gradually and under close monitoring.
B
Discontinue ICS and continue LABA
Hint:
Discontinuing ICS and continuing LABA is not an appropriate step-down option for this patient. This would correspond to step 2 of the asthma treatment algorithm which involves low-dose ICS alone or alternative controllers such as leukotriene receptor antagonists or chromones. However, discontinuing ICS may lead to worsening inflammation, symptoms, lung function, and quality of life due to loss of steroid effects. Therefore, if ICS is discontinued, it should be done gradually and under close monitoring. Furthermore, using LABA alone without concomitant ICS may increase the risk of severe exacerbations or death due to paradoxical bronchoconstriction or masking effects. Therefore, LABA should never be used as monotherapy for asthma.
C
Reduce ICS dose by 50% and continue LABA
Hint:
Reducing ICS dose by 50% and continuing LABA is not an appropriate step-down option for this patient. This would correspond to step 3 of the asthma treatment algorithm which still involves a relatively high dose of ICS plus LABA. Moreover, reducing ICS dose by more than one level at a time may increase the risk of exacerbations or loss of control due to insufficient anti-inflammatory effects. Therefore, if ICS dose is reduced, it should be done gradually by one level at a time every 2–3 months until reaching a low-dose level.
D
Reduce both ICS and LABA doses by 50%
Hint:
Reducing both ICS and LABA doses by 50% is not an appropriate step-down option for this patient. This would correspond to step 2–3 depending on whether low- or medium-dose level was reached after reduction. However, reducing both components simultaneously may increase the risk of exacerbations or loss of control due to insufficient anti-inflammatory and bronchodilator effects. Therefore, if both components are reduced, it should be done sequentially rather than simultaneously, starting with reducing LABA first followed by reducing ICS after achieving good control.
E
Switch to low-dose ICS/formoterol as needed
Question 37 Explanation: 
This patient has well-controlled asthma on medium-dose ICS and LABA, which corresponds to step 4 of the asthma treatment algorithm. Step-down therapy can be considered for patients with at least 3 months of continuous good control of asthma. The goal of step-down therapy is to reduce medication use to the lowest effective dose while maintaining asthma control. According to the Global Initiative for Asthma (GINA), one option for stepping down from step 4 is to switch to low-dose ICS/formoterol as needed. This regimen involves using a combination inhaler containing low-dose ICS (budesonide) and formoterol (a fast-acting LABA) both as maintenance therapy and as reliever therapy instead of a short-acting beta agonist (SABA). This strategy has been shown to reduce exacerbations, improve symptom control, and decrease steroid exposure compared with conventional maintenance therapy with higher doses of ICS/LABA plus SABA as needed. (Review PANRE Blueprint Topic: Asthma)
Question 38
A 45-year-old woman presents to your office with a complaint of frequent headaches. Her blood pressure is found to be 150/95 mmHg on three separate occasions. She has no other medical conditions and takes no medications. Which of the following is the most appropriate initial treatment?
A
ACE inhibitor alone
Hint:
ACE inhibitors help to relax blood vessels by blocking the formation of a hormone that narrows them. They may be used as an initial antihypertensive treatment in the general nonblack population. They are always used in combination with lifestyle modifications.
B
Calcium channel blocker alone
Hint:
Calcium channel blockers help relax blood vessels by preventing calcium from entering them. They may be used as an initial antihypertensive treatment in the general black population. They are always used in combination with lifestyle modifications.
C
Thiazide diuretic alone
Hint:
Thiazide diuretics help remove sodium and water from the body by increasing urine output. They may be used as an initial antihypertensive treatment in the general nonblack and black populations, but only in combination with lifestyle modifications.
D
Thiazide diuretic + ACE inhibitor
Hint:
This is a combination of two drugs that have different mechanisms of action to lower blood pressure. It may be used as an initial antihypertensive treatment in patients with stage 2 hypertension (systolic blood pressure ≥ 160 mm Hg or diastolic blood pressure ≥ 100 mm Hg), but only in combination with lifestyle modifications.
E
Lifestyle modifications
Question 38 Explanation: 
The patient has hypertension, which is defined as a systolic blood pressure of 130 mm Hg or more or a diastolic blood pressure of 80 mm Hg or more. The initial treatment of hypertension should include lifestyle modifications such as salt reduction, moderation of alcohol consumption, and a diet high in vegetables and fruit that is low in added sugars and saturated fats (e.g., DASH diet). These interventions can lower blood pressure by an average of 5 to 20 mm Hg. (Review PANRE Blueprint Topic: Essential Hypertension)
Question 39
A 7-year-old child presents with honey-colored crusting lesions on the face and extremities. The lesions started as small red papules that quickly progressed to vesicles and then ruptured, leaving behind a honey-colored crust. The child has no fever and is otherwise healthy. What is the most appropriate initial treatment for this condition?
A
Topical corticosteroids
Hint:
Topical corticosteroids are not appropriate for impetigo as they can exacerbate the infection by suppressing the immune response and promoting bacterial growth.
B
Oral antihistamines
Hint:
Oral antihistamines are used to treat allergic reactions and are not effective in treating bacterial skin infections such as impetigo.
C
Oral doxycycline
Hint:
Oral doxycycline can be used to treat impetigo, but it is not the initial treatment of choice. Oral antibiotics are reserved for cases of extensive or severe impetigo or for patients who cannot tolerate topical antibiotics.
D
Topical mupirocin
E
Oral acyclovir
Hint:
Oral acyclovir is an antiviral medication used to treat viral skin infections such as herpes simplex virus (HSV). It is not effective in treating bacterial skin infections such as impetigo.
Question 39 Explanation: 
The presentation of honey-colored crusting lesions on the face and extremities in a child is consistent with impetigo, a bacterial skin infection caused by Staphylococcus aureus or Streptococcus pyogenes. The most appropriate initial treatment for impetigo is topical antibiotics such as mupirocin. This is because impetigo is a localized skin infection, and topical antibiotics can effectively treat it without the need for systemic antibiotics. The topical antibiotic is applied to the affected area 2-3 times per day for 5-7 days. (Review PANRE Blueprint Topic: Impetigo)
Question 40
A 32-year-old woman presents to her primary care PA with a history of recurrent depressive episodes during the winter months over the past three years. The patient reports feelings of low mood, lack of interest in activities, increased fatigue, and increased appetite during these episodes. Which of the following treatments would be most appropriate for this patient's condition?
A
Psychotherapy alone
Hint:
Psychotherapy alone is unlikely to be effective in treating SAD, although it may be helpful as an adjunct to medication and light therapy. Therefore, this answer choice is incorrect.
B
Selective serotonin reuptake inhibitor (SSRI) alone
Hint:
SSRIs are effective in treating SAD, but the most effective treatment for SAD is a combination of light therapy and medication. Therefore, this answer choice is not the best option.
C
Light therapy alone
Hint:
Light therapy alone may be effective in some cases, but the combination of light therapy and medication is generally considered to be the most effective treatment for SAD. Therefore, this answer choice is not the best option.
D
Psychotherapy and SSRI
Hint:
The combination of psychotherapy and medication is generally effective in treating major depressive disorder, but for major depressive disorder with seasonal pattern, the most effective treatment is a combination of light therapy and medication. Therefore, this answer choice is not the best option.
E
Light therapy and SSRI
Question 40 Explanation: 
Major depressive disorder with seasonal pattern, also known as seasonal affective disorder (SAD), is a subtype of major depressive disorder that occurs in a seasonal pattern, typically during the fall and winter months. The most effective treatment for SAD is a combination of light therapy and medication. Light therapy involves exposure to bright, artificial light, which can help regulate the body's internal clock and improve mood. Selective serotonin reuptake inhibitors (SSRIs) are also effective in treating SAD by increasing serotonin levels in the brain. Psychotherapy alone is unlikely to be effective in treating SAD, although it may be helpful as an adjunct to medication and light therapy. Light therapy alone may be effective in some cases, but the combination of light therapy and medication is generally considered to be the most effective treatment for SAD.
Question 41
A 45-year-old female presents with fatigue, weight gain, cold intolerance, and constipation. Her thyroid-stimulating hormone (TSH) level is elevated, and her free thyroxine (T4) level is decreased. Which of the following is the most likely cause of her hypothyroidism?
A
Iodine deficiency
Hint:
Iodine deficiency can lead to hypothyroidism, but it is a relatively rare cause in the United States due to widespread iodine supplementation.
B
Pituitary adenoma
Hint:
Pituitary adenoma is a rare cause of hypothyroidism and is more commonly associated with hyperthyroidism due to excess thyroid-stimulating hormone (TSH) secretion.
C
Hashimoto's thyroiditis
D
Thyroid cancer
Hint:
Thyroid cancer is not a common cause of hypothyroidism. While some types of thyroid cancer can lead to decreased thyroid hormone production, the majority of thyroid cancers do not affect thyroid function.
E
Graves' disease
Hint:
Graves' disease is a common cause of hyperthyroidism, not hypothyroidism. In Graves' disease, the immune system stimulates the thyroid gland to produce excess thyroid hormone, leading to hyperthyroidism symptoms such as weight loss, heat intolerance, and tachycardia.
Question 41 Explanation: 
Hashimoto's thyroiditis is the most common cause of hypothyroidism in the United States. It is an autoimmune disorder in which the body's immune system attacks the thyroid gland, leading to inflammation and damage. This results in decreased production of thyroid hormone and hypothyroidism. The symptoms of hypothyroidism, including fatigue, weight gain, cold intolerance, and constipation, are common in Hashimoto's thyroiditis. The elevated TSH and decreased free T4 levels seen in this patient are also consistent with this diagnosis. (Review PANRE Blueprint Topic: Hypothyroidism)
Question 42

A 25-year-old woman presents to your clinic with a 2-day history of dysuria, frequency, urgency, and suprapubic pain. She denies fever, chills, flank pain, vaginal discharge, or hematuria. She is sexually active with one male partner and uses oral contraceptives. Her last menstrual period was 2 weeks ago. Her vital signs are normal. Urinalysis shows positive leukocyte esterase and nitrites. Urine culture is pending. What is the most appropriate initial treatment for this patient?

A
Ciprofloxacin 500 mg orally twice daily for 3 days
Hint:
Ciprofloxacin is a fluoroquinolone antibiotic that has good activity against most gram-negative bacteria causing UTIs. However, it is not a first-line agent for uncomplicated cystitis due to increasing resistance rates (>20%) and potential adverse effects such as tendon rupture, QT prolongation, and Clostridioides difficile infection. Fluoroquinolones should be reserved for complicated UTIs or patients with allergies or contraindications to other agents.
B
Levofloxacin 750 mg orally once daily for 5 days
Hint:
This option is another fluoroquinolone antibiotic that has the same disadvantages as ciprofloxacin and is not a first-line agent for uncomplicated cystitis.
C
Nitrofurantoin 100 mg orally twice daily for 5 days
D
Trimethoprim-sulfamethoxazole (TMP-SMX) 160/800 mg orally twice daily for 3 days
Hint:
This option is a combination antibiotic that was previously a first-line agent for uncomplicated cystitis. However, it is now recommended only if the local resistance rate of E. coli to TMP-SMX is <20% and the patient has no risk factors for resistance (such as recent antibiotic use, travel to areas with high resistance, or health care exposure) . TMP-SMX also has more adverse effects than nitrofurantoin or fosfomycin such as hypersensitivity reactions, hematologic abnormalities, hyperkalemia, and C. difficile infection. Additionally, TMP-SMX should be avoided in patients with sulfa allergy or G6PD deficiency.
E
No antibiotic therapy until urine culture results are available
Hint:
This is inappropriate for patients with symptomatic acute cystitis who have positive urinalysis findings suggestive of bacterial infection. Delaying antibiotic therapy may increase the risk of complications such as pyelonephritis (upper UTI involving the kidney), urosepsis (bacteremia from UTI), or renal scarring. Empirical antibiotic therapy should be initiated promptly based on local resistance patterns and adjusted if needed when urine culture results are available.
Question 42 Explanation: 
The patient has a clinical presentation consistent with uncomplicated acute bacterial cystitis, which is a lower urinary tract infection (UTI) involving the bladder. Uncomplicated cystitis refers to an infection in otherwise healthy women who have no structural or functional abnormalities of the urinary tract. The most common causative organisms are Escherichia coli (75-95%), followed by Staphylococcus saprophyticus (5-15%), Klebsiella pneumoniae, Proteus mirabilis, and Enterococcus faecalis. The diagnosis of acute cystitis can be made based on symptoms and urinalysis findings of pyuria (leukocyte esterase positive) and bacteriuria (nitrite positive). Urine culture is not routinely indicated for uncomplicated cystitis unless symptoms persist or recur after treatment.The initial treatment of choice for uncomplicated acute cystitis is empirical antibiotic therapy based on local resistance patterns. The recommended first-line agents are nitrofurantoin monohydrate/macrocrystals (Macrobid) 100 mg orally twice daily for 5 days or fosfomycin trometamol (Monurol) single dose of three grams dissolved in water. These agents have low resistance rates (<10%) and minimal collateral damage to the gut flora. Nitrofurantoin has excellent activity against E. coli and S. saprophyticus but does not cover Proteus or Pseudomonas species. Fosfomycin has broad-spectrum activity against most gram-negative and gram-positive bacteria including E. coli producing extended-spectrum beta-lactamases (ESBLs). (Review PANRE Blueprint Topic: Cystitis )
Question 43
What is the typical time frame for the onset of postpartum depression?
A
During pregnancy
Hint:
See C for explanation
B
Within 24 hours after delivery
Hint:
See C for explanation
C
Within 1-2 weeks after delivery
D
Within 6 months after delivery
Hint:
See C for explanation
E
More than a year after delivery
Hint:
See C for explanation
Question 43 Explanation: 
Postpartum depression (PPD) is a common condition that affects around 15-20% of women after giving birth. Postpartum depression typically occurs within the first 1-2 weeks after delivery. However, it can also occur during pregnancy or up to 6 months after delivery. It is important for healthcare providers to screen women for postpartum depression during routine visits throughout the first year after delivery.
Once you are finished, click the button below. Any items you have not completed will be marked incorrect. Get Results
There are 43 questions to complete.
List
Return
Shaded items are complete.
12345
678910
1112131415
1617181920
2122232425
2627282930
3132333435
3637383940
414243End
Return